Hallo zusammen,

ich bräuchte einen Ansatz zur Lösung folgender Aufgabe.

Ich müsste wissen wie ich auf den Winkel Alpha 1 komme. Den weiteren Rechenweg kann ich von vorherigen Aufgaben.

Danke für eure Hilfe.